Last visit was: 26 Apr 2024, 13:34 It is currently 26 Apr 2024, 13:34

Close
GMAT Club Daily Prep
Thank you for using the timer - this advanced tool can estimate your performance and suggest more practice questions. We have subscribed you to Daily Prep Questions via email.

Customized
for You

we will pick new questions that match your level based on your Timer History

Track
Your Progress

every week, we’ll send you an estimated GMAT score based on your performance

Practice
Pays

we will pick new questions that match your level based on your Timer History
Not interested in getting valuable practice questions and articles delivered to your email? No problem, unsubscribe here.
Close
Request Expert Reply
Confirm Cancel
SORT BY:
Date
Tags:
Show Tags
Hide Tags
User avatar
Manager
Manager
Joined: 02 Nov 2014
Posts: 152
Own Kudos [?]: 431 [0]
Given Kudos: 75
GMAT Date: 08-04-2015
Send PM
avatar
Intern
Intern
Joined: 13 Sep 2015
Posts: 13
Own Kudos [?]: 31 [0]
Given Kudos: 239
Send PM
User avatar
Manager
Manager
Joined: 02 Nov 2014
Posts: 152
Own Kudos [?]: 431 [0]
Given Kudos: 75
GMAT Date: 08-04-2015
Send PM
avatar
Intern
Intern
Joined: 14 Jun 2016
Posts: 13
Own Kudos [?]: 18 [0]
Given Kudos: 50
Location: United States
GMAT 1: 700 Q48 V37
WE:Engineering (Pharmaceuticals and Biotech)
Send PM
Re: Denoma, a major consumer-electronics maker, had a sizeable decline in [#permalink]
vaibhav123 wrote:
ChrisLele wrote:
So let’s take apart the argument.

Denoma (D.), a consumer-electronic maker, has seen a decrease in sales in the past year. However, D. provides products to retailers, which saw an increase in the sale of D.’s products. How to account for this?

Well, maybe D.’s profits are not solely determined by the number of its products users are able to sell. Maybe, D. creates parts for products that do not have the D. label on them.

For example, let’s say retailers sold D.s new line of home stereos. Units flew off the shelf. However, D. makes a number of parts for many televisions, which do not have the D. label (let’s say for Sony, Panasonic and the rest) If most of D.’s profits are determined by the sale of televisions, then if televisions had a bad year, D.’s profits will decline. This is the case, even with retailers selling plenty of D’s stereos.

The only answer choice that captures this logic is (C).

Let me know that if that helped!



What is wrong with A)


Their advertisement is cutting into their profit but not necessarily affects their revenue.
Intern
Intern
Joined: 01 Sep 2016
Posts: 35
Own Kudos [?]: 39 [0]
Given Kudos: 93
Send PM
Re: Denoma, a major consumer-electronics maker, had a sizeable decline in [#permalink]
Thanks for the crisp explanation :)

Regards,
Abhishek SInha

thevenus wrote:
Ofcourse C

Let's keep it short and simple, the argument states that:
-Overall revenue of company has declined BUT
-The retail sales has done good business
It means that, the overall decremented revenue is not directly related to the retail sales ONLY.Something else is responsible for the overall sales/revenue decline.

Only C states that a substantial revenue is dependent on something else too ( "A significant proportion of Denoma’s revenue comes from making components for other consumer-electronics manufacturers.")
Senior Manager
Senior Manager
Joined: 05 Jul 2017
Posts: 457
Own Kudos [?]: 724 [0]
Given Kudos: 294
Location: India
GMAT 1: 700 Q49 V36
GPA: 4
Send PM
Re: Denoma, a major consumer-electronics maker, had a sizeable decline in [#permalink]
Hey nightblade354

Need your help. I am confused between option A and C

I discarded option C for the below reason and went ahead with option A. Let me know yourthoughts

Option C : - A significant proportion of Denoma’s revenue comes from making components for other consumer-electronics manufacturers.

Components are sold by denoma to other manufacturers. It means, denoma already received the money for the same. If the other manufacturers didnt sell them then this situation shouldnt be affecting denoma. We will have to assume that denoma will receive the money for the components they made only if the other manufacturers sold their respective products. Hence I ruled out C
Current Student
Joined: 31 Jul 2017
Status:He came. He saw. He conquered. -- Going to Business School -- Corruptus in Extremis
Posts: 1734
Own Kudos [?]: 5743 [2]
Given Kudos: 3054
Location: United States (MA)
Concentration: Finance, Economics
Send PM
Re: Denoma, a major consumer-electronics maker, had a sizeable decline in [#permalink]
2
Kudos
Expert Reply
pikolo2510, let's break the argument down.

Denoma, a major consumer-electronics maker, had a sizeable decline in sales revenue for its most recent fiscal year. This result appears surprising, because electronics retailers report that although their overall sales were considerably lower than in the previous year, their sales revenue from Denoma models actually grew, largely thanks to some innovative and popular models that Denoma introduced.

Which of the following, if true, does most to explain the apparently surprising result?

(A) Because of the need to educate the public about its new models’ capabilities, Denoma’s advertising spending was higher than normal over the period. -- What does the last portion mean when it says "higher than normal"? Does this mean by a penny, or by $50 billion? We cannot make this assumption and thus doesn't help our argument.

(C) A significant proportion of Denoma’s revenue comes from making components for other consumer-electronics manufacturers. -- Ah, so if their profits are deriven from something else, this would resolve the paradox.

Does this help?
Manager
Manager
Joined: 28 Jun 2018
Posts: 96
Own Kudos [?]: 221 [0]
Given Kudos: 329
Location: Bouvet Island
GMAT 1: 670 Q50 V31
GMAT 2: 640 Q47 V30
GMAT 3: 700 Q49 V36
GMAT 4: 490 Q39 V18
GPA: 4
Send PM
Re: Denoma, a major consumer-electronics maker, had a sizeable decline in [#permalink]
nightblade354 wrote:
pikolo2510, let's break the argument down.

Denoma, a major consumer-electronics maker, had a sizeable decline in sales revenue for its most recent fiscal year. This result appears surprising, because electronics retailers report that although their overall sales were considerably lower than in the previous year, their sales revenue from Denoma models actually grew, largely thanks to some innovative and popular models that Denoma introduced.

Which of the following, if true, does most to explain the apparently surprising result?

(A) Because of the need to educate the public about its new models’ capabilities, Denoma’s advertising spending was higher than normal over the period. -- What does the last portion mean when it says "higher than normal"? Does this mean by a penny, or by $50 billion? We cannot make this assumption and thus doesn't help our argument.

(C) A significant proportion of Denoma’s revenue comes from making components for other consumer-electronics manufacturers. -- Ah, so if their profits are deriven from something else, this would resolve the paradox.

Does this help?


Hello nightblade354! :)

I had a small doubt!
We are mainly concerned with "Sales Revenue".
Do we really need to bother about an option that talks about advertising? Does it matter what happens to the revenue? Wether they spend it on advertising or something else?

I agree with your above explanation, but was curious if we could eliminate the option in any other way!

Posted from my mobile device
Current Student
Joined: 31 Jul 2017
Status:He came. He saw. He conquered. -- Going to Business School -- Corruptus in Extremis
Posts: 1734
Own Kudos [?]: 5743 [1]
Given Kudos: 3054
Location: United States (MA)
Concentration: Finance, Economics
Send PM
Re: Denoma, a major consumer-electronics maker, had a sizeable decline in [#permalink]
Expert Reply
Hi blitzkriegxX,

We cannot eliminate off of the premise you just mentioned. A failed advertising campaign could be expensive and worthless; this would resolve the paradox (revenue down, but sales up).
Intern
Intern
Joined: 04 May 2019
Posts: 20
Own Kudos [?]: 53 [0]
Given Kudos: 12
Send PM
Re: Denoma, a major consumer-electronics maker, had a sizeable decline in [#permalink]
Why is A wrong? since D's advertising expenses were higher, its net profit was less and hence revenue was less. Doesnt it resolve the conflict?
Intern
Intern
Joined: 09 Jan 2018
Posts: 19
Own Kudos [?]: 16 [0]
Given Kudos: 9
Send PM
Re: Denoma, a major consumer-electronics maker, had a sizeable decline in [#permalink]
d3thknell wrote:
the argument talks about an consumer electronics maker Denoma,which had a sizeable decline in sales revenue for that particular year. this result comes as a surprise to everyone because according to retailers of electronic goods( the people who sell electronics), although there was a decline in overall sales of electronic goods but the goods manufactured by denoma had increased sales revenue compared to last year. this was largely due to innovation and popularity of newly introduced models of denoma.

so what we have from the argument is basically
-increase in sales revenue from the models manufactured by denoma.
-overall decline in sales of electronic goods.
-overall decline in sales revenue for Denoma.
how can one explain this effect?
if we can assume denoma is also into manufacturing of parts for other electronic goods manufacturers and the revenue from these parts is is much more significant compared to revenue from denoma models, then the above three effects can be explained.


I feel like option C requires us to make an additional assumption.
Revenue = no. of units sold x price per unit.
Revenue decrease could be due to number of units sold decrease as mentioned in option C.
But we are assuming that revenue from components manufacture for other consumer electronics decreased.
We do not know if other consumer decreased their buying or consumption of Denoma componenets.

If we are making additional assumptions, E is also a strong contender because it says retailers sold them at discounted price.
we can assume - Denoma also had to sell them at a lesser price, which will reduce revenue.

Can anyone please explain if my reasoning is on the right lines? Why am i wrong? OR How do you approach it?
GMAT Club Verbal Expert
Joined: 13 Aug 2009
Status: GMAT/GRE/LSAT tutors
Posts: 6923
Own Kudos [?]: 63673 [1]
Given Kudos: 1774
Location: United States (CO)
GMAT 1: 780 Q51 V46
GMAT 2: 800 Q51 V51
GRE 1: Q170 V170

GRE 2: Q170 V170
Send PM
Re: Denoma, a major consumer-electronics maker, had a sizeable decline in [#permalink]
1
Kudos
Expert Reply
payalkhndlwl wrote:
Why is A wrong? since D's advertising expenses were higher, its net profit was less and hence revenue was less. Doesnt it resolve the conflict?

We're asked which choice does the most to explain the apparently surprising result (or as you put it, the conflict). But what is the conflict we're trying to explain?

Well, the result is that Denoma had a sizeable decline in sales revenue during its most recent fiscal year. We are not looking at net profit, which is typically calculated as gross income (including sales revenue, but potentially also including other sources of income) minus the cost of goods and services.

The conflict here is that Denoma had a sizeable decline in sales revenue over the fiscal year — but in the same year, retailers saw an increase in sales revenue from Denoma models.

Choice (A) states that Denoma's advertising spend was higher than normal over the period in question. But that doesn't explain the conflict that we're concerned with:

  • Again, this question is not about net profit.
  • Knowing that there was higher advertising spend doesn't give us any new information on why Denoma's overall sales revenue declined.

As nightblade354 and others have explained, choice (C) more directly addresses the conflict. If a significant proportion of Denoma's revenue comes from selling components to other manufacturers, then a decline in these sales to manufacturers could result in a decline in overall sales revenue — even at the same time as sales revenue from retailers grows.

This information does more to resolve the conflict than any other answer choice.

I hope this helps!
Senior Manager
Senior Manager
Joined: 27 Mar 2017
Posts: 273
Own Kudos [?]: 76 [0]
Given Kudos: 406
Location: Saudi Arabia
GMAT 1: 700 Q47 V39
GPA: 3.36
Send PM
Denoma, a major consumer-electronics maker, had a sizeable decline in [#permalink]
GMATNinja wrote:
payalkhndlwl wrote:
Why is A wrong? since D's advertising expenses were higher, its net profit was less and hence revenue was less. Doesnt it resolve the conflict?

We're asked which choice does the most to explain the apparently surprising result (or as you put it, the conflict). But what is the conflict we're trying to explain?

Well, the result is that Denoma had a sizeable decline in sales revenue during its most recent fiscal year. We are not looking at net profit, which is typically calculated as gross income (including sales revenue, but potentially also including other sources of income) minus the cost of goods and services.

The conflict here is that Denoma had a sizeable decline in sales revenue over the fiscal year — but in the same year, retailers saw an increase in sales revenue from Denoma models.

Choice (A) states that Denoma's advertising spend was higher than normal over the period in question. But that doesn't explain the conflict that we're concerned with:

  • Again, this question is not about net profit.
  • Knowing that there was higher advertising spend doesn't give us any new information on why Denoma's overall sales revenue declined.

As nightblade354 and others have explained, choice (C) more directly addresses the conflict. If a significant proportion of Denoma's revenue comes from selling components to other manufacturers, then a decline in these sales to manufacturers could result in a decline in overall sales revenue — even at the same time as sales revenue from retailers grows.

This information does more to resolve the conflict than any other answer choice.

I hope this helps!


Hi,

Compared to (E), doesn't (C) require us to make an extra assumption that the revenue of the other manufacturers declined as well ?

So what if Denoma's revenue comes from other manufacturers ? We don't know what happened to their revenue. That should be largely out of scope.

On the other hand, (E) states that older models (since stimulus clearly states that increase in sales was from newer models) were sold at a significantly lower price as compared to their original cost. The stimulus even states that 'largely' the number of units sold were newer. So this is a much smaller leap in assumption that enough number of older units were sold to have a sizeable impact on the overall revenue. (C) on the other hand asks a big leap in assumption that that the combined revenue of other sources decreased. While we can make that assumption, what's stopping us from assuming the required assumption for (E) i.e. enough older units were sold to make a considerable impact on revenue.

(C) would have made more sense, had the stimulus hinted that overall market saw decreasing revenues.

(C) isn't a no-brainer asnwer, or is it ? What it appears to refer to can easily be termed out-of-scope, no ?

Will appreciate your input.
GMAT Club Verbal Expert
Joined: 13 Aug 2009
Status: GMAT/GRE/LSAT tutors
Posts: 6923
Own Kudos [?]: 63673 [4]
Given Kudos: 1774
Location: United States (CO)
GMAT 1: 780 Q51 V46
GMAT 2: 800 Q51 V51
GRE 1: Q170 V170

GRE 2: Q170 V170
Send PM
Re: Denoma, a major consumer-electronics maker, had a sizeable decline in [#permalink]
3
Kudos
1
Bookmarks
Expert Reply
altairahmad wrote:
GMATNinja wrote:
payalkhndlwl wrote:
Why is A wrong? since D's advertising expenses were higher, its net profit was less and hence revenue was less. Doesnt it resolve the conflict?

We're asked which choice does the most to explain the apparently surprising result (or as you put it, the conflict). But what is the conflict we're trying to explain?

Well, the result is that Denoma had a sizeable decline in sales revenue during its most recent fiscal year. We are not looking at net profit, which is typically calculated as gross income (including sales revenue, but potentially also including other sources of income) minus the cost of goods and services.

The conflict here is that Denoma had a sizeable decline in sales revenue over the fiscal year — but in the same year, retailers saw an increase in sales revenue from Denoma models.

Choice (A) states that Denoma's advertising spend was higher than normal over the period in question. But that doesn't explain the conflict that we're concerned with:

  • Again, this question is not about net profit.
  • Knowing that there was higher advertising spend doesn't give us any new information on why Denoma's overall sales revenue declined.

As nightblade354 and others have explained, choice (C) more directly addresses the conflict. If a significant proportion of Denoma's revenue comes from selling components to other manufacturers, then a decline in these sales to manufacturers could result in a decline in overall sales revenue — even at the same time as sales revenue from retailers grows.

This information does more to resolve the conflict than any other answer choice.

I hope this helps!


Hi,

Compared to (E), doesn't (C) require us to make an extra assumption that the revenue of the other manufacturers declined as well ?

So what if Denoma's revenue comes from other manufacturers ? We don't know what happened to their revenue. That should be largely out of scope.

On the other hand, (E) states that older models (since stimulus clearly states that increase in sales was from newer models) were sold at a significantly lower price as compared to their original cost. The stimulus even states that 'largely' the number of units sold were newer. So this is a much smaller leap in assumption that enough number of older units were sold to have a sizeable impact on the overall revenue. (C) on the other hand asks a big leap in assumption that that the combined revenue of other sources decreased. While we can make that assumption, what's stopping us from assuming the required assumption for (E) i.e. enough older units were sold to make a considerable impact on revenue.

(C) would have made more sense, had the stimulus hinted that overall market saw decreasing revenues.

(C) isn't a no-brainer asnwer, or is it ? What it appears to refer to can easily be termed out-of-scope, no ?

Will appreciate your input.

We're not looking for a "no-brainer answer." We are selecting the answer choice that does most to explain the apparently surprising result.

This doesn't require us to confirm specific assumptions, as you've been trying to do in your analysis. We only need to identify the choice that does the most to make this surprising result logically believable.

The prompt tells us Denoma had a sizeable decline in sales revenue for its most recent fiscal year.. This is quite a broad statement by definition, so it covers anything that Denoma has sold to the "overall market," as you put it — including the sales of components to the consumer-electronics market. I'm not sure if this is what you meant when comparing the scope of (C) to the scope of the prompt, but in any case, it would be a mistake to assume that this decline in sales revenue only applies to specific product verticals.

Taking one more look at choice (E):

Quote:
(E) During the period, consumer-electronics retailers sold remaining units of Denoma’s superseded models at prices that were deeply discounted from those models’ original prices.

You're right that this is a very concrete story. But is it the story we care about?

Remember, we want the choice that does the most to raise logically valid possibilities for a sizable decline in sales revenue.

Putting (C) and (E) head to head, here's what we've got:

  • (C) confirms that there's an entirely different and significant portion of Denoma's sales portfolio (comopnents), which isn't even covered by the prompt. This raises the logical possibility that a decline in component sales accounts for the decline in overall sales. Sure, we don't have proof that this happened, but the scale of sales decline suggested in this choice matches the scale of the paradox we're trying to explain. That makes the paradox more logically believable.
  • (E) confirms that some portion of back stock of outdated models were sold at a discount alongside their innovative and popular models. This only tells us that... retailers sold off some old models at a cheaper price in addition to Denoma's great new models. Could this alone can explain why Denoma saw a sizable decline in overall sales revenue? Maaaaybe, but compared to the possible swing in revenue suggested in (C), this is weak sauce. It's a narrow data point covering what seems like a small proportion of sales — which does very little to make Denoma's overall decline in sales revenue more logically believable.

I hope this helps!
VP
VP
Joined: 14 Feb 2017
Posts: 1115
Own Kudos [?]: 2164 [1]
Given Kudos: 368
Location: Australia
Concentration: Technology, Strategy
GMAT 1: 560 Q41 V26
GMAT 2: 550 Q43 V23
GMAT 3: 650 Q47 V33
GMAT 4: 650 Q44 V36
GMAT 5: 600 Q38 V35
GMAT 6: 710 Q47 V41
WE:Management Consulting (Consulting)
Send PM
Re: Denoma, a major consumer-electronics maker, had a sizeable decline in [#permalink]
1
Kudos
(E) During the period, consumer-electronics retailers sold remaining units of Denoma’s superseded models at prices that were deeply discounted from those models’ original prices.

E is incorrect. We are told that retailers report that Denomer models' sales are up this year. The paradox is that Denoma's sales (Denoma is a manufacturer) are down despite retailers' report. Denoma's sales comprise products sold to retailers and since we are already told that retailer's sales of denomer models are up, it doesn't matter whether some denoma models were discounted or sold at full price.

C on the other hand is correct and logically links whats going on. Retailer's sales of other non-denoma models are down. If those models are comprised of Denoma parts, then the demand for denoma parts would be down also, and since denoma makes a 'large proportion' of its revenue from such sales it would stand to justify that Denoma's sales this year are down despite the increase in sales of Denoma branded models.
Re: Denoma, a major consumer-electronics maker, had a sizeable decline in [#permalink]
1
Bookmarks
Quote:
Denoma, a major consumer-electronics maker, had a sizeable decline in sales revenue for its most recent fiscal year. This result appears surprising, because electronics retailers report that although their overall sales were considerably lower than in the previous year, their sales revenue from Denoma models actually grew, largely thanks to some innovative and popular models that Denoma introduced.

Which of the following, if true, does most to explain the apparently surprising result?

(A) Because of the need to educate the public about its new models??? capabilities, Denoma???s advertising spending was higher than normal over the period.
(B) For the period at issue, Denoma???s major competitors reported declines in revenue that were, in percentage terms, greater than Denoma???s.
(C) A significant proportion of Denoma???s revenue comes from making components for other consumer-electronics manufacturers.
(D) Unlike some of its major competitors, Denoma has no lines of business outside consumer electronics to provide revenue when retail sales of consumer electronics are weak.
(E) During the period, consumer-electronics retailers sold remaining units of Denoma???s superseded models at prices that were deeply discounted from those models??? original prices.

Hello,
Honorable experts,
q1:
from choice C:
who will confirm that OTHER side's sales revenue is going down?
q2:
a bit curiosity on the following question...
what if the "a major consumer-electronics maker" has been removed from the argument?
Can we still confirm choice C as legit one?
Thanks__
Manager
Manager
Joined: 15 Jul 2014
Posts: 92
Own Kudos [?]: 103 [0]
Given Kudos: 233
Location: India
Concentration: Marketing, Technology
Send PM
Denoma, a major consumer-electronics maker, had a sizeable decline in [#permalink]
Asad wrote:
Quote:
Denoma, a major consumer-electronics maker, had a sizeable decline in sales revenue for its most recent fiscal year. This result appears surprising, because electronics retailers report that although their overall sales were considerably lower than in the previous year, their sales revenue from Denoma models actually grew, largely thanks to some innovative and popular models that Denoma introduced.

Which of the following, if true, does most to explain the apparently surprising result?

(A) Because of the need to educate the public about its new models??? capabilities, Denoma???s advertising spending was higher than normal over the period.
(B) For the period at issue, Denoma???s major competitors reported declines in revenue that were, in percentage terms, greater than Denoma???s.
(C) A significant proportion of Denoma???s revenue comes from making components for other consumer-electronics manufacturers.
(D) Unlike some of its major competitors, Denoma has no lines of business outside consumer electronics to provide revenue when retail sales of consumer electronics are weak.
(E) During the period, consumer-electronics retailers sold remaining units of Denoma???s superseded models at prices that were deeply discounted from those models??? original prices.

Hello,
Honorable experts,
q1:
from choice C:
who will confirm that OTHER side's sales revenue is going down?
q2:
a bit curiosity on the following question...
what if the "a major consumer-electronics maker" has been removed from the argument?
Can we still confirm choice C as legit one?
Thanks__

Q1: This is given in the question stem. Other side's revenue is already less than that of previous year. Consider this scenario. Last year, D's electronic retailers' ( ones who use D's components in their products) revenue was 100$. This year, it was 80$. Now, comparatively, D would get less revenues because of the decline in considerably lower consumer electronic sales. Though Denoma models' revenue grew, it did not outgrew to compensate the loss from revenues.
Q2: Yes. We could still consider C as legit. The fact that Denoma is a major consumer-electronics maker doesn't contribute anything to the conclusion. The contributing factor is Denoma manufactures both in-house products and components for other MNCs. Hope this helps.

Please let me know whether this works for you!
VP
VP
Joined: 14 Aug 2019
Posts: 1378
Own Kudos [?]: 846 [0]
Given Kudos: 381
Location: Hong Kong
Concentration: Strategy, Marketing
GMAT 1: 650 Q49 V29
GPA: 3.81
Send PM
Denoma, a major consumer-electronics maker, had a sizeable decline in [#permalink]
Quote:
Denoma, a major consumer-electronics maker, had a sizeable decline in sales revenue for its most recent fiscal year. This result appears surprising, because electronics retailers report that although their overall sales were considerably lower than in the previous year, their sales revenue from Denoma models actually grew, largely thanks to some innovative and popular models that Denoma introduced.

Which of the following, if true, does most to explain the apparently surprising result?


Need to explain:
1. Had a sizeable decline in sales revenue for its most recent fiscal yea
2. Retailers overall sales were considerably lower than in the previous year
3. Sales revenue from Denoma models actually grew because of new models

Quote:
(A) Because of the need to educate the public about its new models’ capabilities, Denoma’s advertising spending was higher than normal over the period.

Explains: 1 (decreased because of advertising)
2: doesn’t explain (why overall revenue fall then?)
3: doesn’t contradict
Reject because it could not explain why overall revenue of retailers fell


Quote:
(B) For the period at issue, Denoma’s major competitors reported declines in revenue that were, in percentage terms, greater than Denoma’s.

Doesn’t contradict: 1
Doesn’t contradict: 2
Doesn’t explain: 3 (how revenue from Denoma increase then)
Reject: because it couldn’t explain then why sales of Denoma of new models didn’t increase

Quote:
(C) A significant proportion of Denoma’s revenue comes from making components for other consumer-electronics manufacturers.

1. Doesn’t contradict why overall denoma revenue fall then?
2. Doesn’t contradict why overall revenue of retailers fall then?
3. Doesn’t explain how still the revenue from Denoma increased?
Reject: because it couldn’t explain then why sales of Denoma of new models didn’t increase
what's new in this year than previous year. If sales of consumers manufacturers fall then D's revenue also fall.
This option come out to be similar to B

Quote:
(D) unlike some of its major competitors, Denoma has no lines of business outside consumer electronics to provide revenue when retail sales of consumer electronics are weak.

1; Explains: Total revenue fall for Denoma because consumer electronics are weak
2; Explains: Overall revenue of retailers fall because retail sales of consumer electronics are weak.
3; doesn’t contradict: in weak sales , their sales revenue from Denoma models grew due to some new models of Denoma.

Quote:
(E) During the period, consumer-electronics retailers sold remaining units of Denoma’s superseded models at prices that were deeply discounted from those models’ original prices.

1; doesn’t contradict
2; contradict: why overall revenue fall then? Doesn’t explain
3; explains: revenue from Denoma increase

Reject : it couldn’t explain anything about revenue w.r.t Denoma

Hi VeritasKarishma GMATNinja
i am lost in B ,C and D. Can you please check what's wrong with my reasoning in above explanation so that i can avoid it thereafter.

please suggest
Tutor
Joined: 16 Oct 2010
Posts: 14831
Own Kudos [?]: 64940 [3]
Given Kudos: 427
Location: Pune, India
Send PM
Re: Denoma, a major consumer-electronics maker, had a sizeable decline in [#permalink]
3
Kudos
Expert Reply
gmatpunjabi wrote:
Denoma, a major consumer-electronics maker, had a sizeable decline in sales revenue for its most recent fiscal year. This result appears surprising, because electronics retailers report that although their overall sales were considerably lower than in the previous year, their sales revenue from Denoma models actually grew, largely thanks to some innovative and popular models that Denoma introduced.

Which of the following, if true, does most to explain the apparently surprising result?

(A) Because of the need to educate the public about its new models’ capabilities, Denoma’s advertising spending was higher than normal over the period.

(B) For the period at issue, Denoma’s major competitors reported declines in revenue that were, in percentage terms, greater than Denoma’s.

(C) A significant proportion of Denoma’s revenue comes from making components for other consumer-electronics manufacturers.

(D) Unlike some of its major competitors, Denoma has no lines of business outside consumer electronics to provide revenue when retail sales of consumer electronics are weak.

(E) During the period, consumer-electronics retailers sold remaining units of Denoma’s superseded models at prices that were deeply discounted from those models’ original prices.


This is a good question which means you need to read it closely to get to the answer but the answer will be unambiguous.

- Denoma, an electronics manufacturer, had a sizeable decline in sales revenue
- Electronics retailers report that although their overall sales were lower, their sales revenue from Denoma models actually grew because of new good models of Denoma.

How do we explain this paradox?
The revenue for Denoma decreased but for retailers it increased from Denoma products.

(A) Because of the need to educate the public about its new models’ capabilities, Denoma’s advertising spending was higher than normal over the period.

We are only talking about revenue, not profit so cost is irrelevant.

(B) For the period at issue, Denoma’s major competitors reported declines in revenue that were, in percentage terms, greater than Denoma’s.

We know that Denoma's competitors most likely reported declines (because retailers reported declines on other manufacturer products). Doesn't help tell us why Denoma reported declines.

(C) A significant proportion of Denoma’s revenue comes from making components for other consumer-electronics manufacturers.

So Denoma's revenue depends on two things (at least) - its own products and making components for others. Its own product sold well, but others' products did not and hence that revenue would have decreased for Denoma. So overall, it saw a decrease in revenue though retailers sold Denoma products well.
Explains the paradox.

(D) Unlike some of its major competitors, Denoma has no lines of business outside consumer electronics to provide revenue when retail sales of consumer electronics are weak.

Irrelevant. Denoma's electronics sales was not weak. We want to explain why the retailers saw increase in revenue from Denoma electronics products but Denoma saw an overall decrease in revenue.

(E) During the period, consumer-electronics retailers sold remaining units of Denoma’s superseded models at prices that were deeply discounted from those models’ original prices.

Retailers, though they sold some units are deep discounts, still saw an increase in revenue from Denoma. Why did Denoma not see an increase in revenue too? This option does not explain it.

Answer (C)
VP
VP
Joined: 14 Aug 2019
Posts: 1378
Own Kudos [?]: 846 [0]
Given Kudos: 381
Location: Hong Kong
Concentration: Strategy, Marketing
GMAT 1: 650 Q49 V29
GPA: 3.81
Send PM
Re: Denoma, a major consumer-electronics maker, had a sizeable decline in [#permalink]
VeritasKarishma wrote:
gmatpunjabi wrote:
Denoma, a major consumer-electronics maker, had a sizeable decline in sales revenue for its most recent fiscal year. This result appears surprising, because electronics retailers report that although their overall sales were considerably lower than in the previous year, their sales revenue from Denoma models actually grew, largely thanks to some innovative and popular models that Denoma introduced.

Which of the following, if true, does most to explain the apparently surprising result?

(A) Because of the need to educate the public about its new models’ capabilities, Denoma’s advertising spending was higher than normal over the period.

(B) For the period at issue, Denoma’s major competitors reported declines in revenue that were, in percentage terms, greater than Denoma’s.

(C) A significant proportion of Denoma’s revenue comes from making components for other consumer-electronics manufacturers.

(D) Unlike some of its major competitors, Denoma has no lines of business outside consumer electronics to provide revenue when retail sales of consumer electronics are weak.

(E) During the period, consumer-electronics retailers sold remaining units of Denoma’s superseded models at prices that were deeply discounted from those models’ original prices.




(E) During the period, consumer-electronics retailers sold remaining units of Denoma’s superseded models at prices that were deeply discounted from those models’ original prices.

Retailers, though they sold some units are deep discounts, still saw an increase in revenue from Denoma. Why did Denoma not see an increase in revenue too? This option does not explain it.

Answer (C)


A clarification:
Quote:
(E) During the period, consumer-electronics retailers sold remaining units of Denoma’s superseded models at prices that were deeply discounted from those models’ original prices.

Retailers, though they sold some units are deep discounts, still saw an increase in revenue from Denoma. Why did Denoma not see an increase in revenue too? This option does not explain it.


If retailers have bought products in advance then after seeling at discount rates, retailers may see hike in revenue from D models but D may not see the increase in its revenue this time.

Sometimes these kind of thoughts takes time,
We should reject such thinking: because we need to assume extra steps to reach a conclusion?

Please suggest VeritasKarishma
GMAT Club Bot
Re: Denoma, a major consumer-electronics maker, had a sizeable decline in [#permalink]
   1   2   3   
Moderators:
GMAT Club Verbal Expert
6923 posts
GMAT Club Verbal Expert
238 posts
CR Forum Moderator
832 posts

Powered by phpBB © phpBB Group | Emoji artwork provided by EmojiOne